Which one of the following is NOT allowed for an issue of the newsletter?

christina on November 2, 2020

Answer choice D

Naz says in the question explanation video that you have to have the features not in slot 1 be M to avoid violating rule 2. Isn’t it also possible to have those features be F if you put T in slot 1?

Replies
Create a free account to read and take part in forum discussions.

Already have an account? log in

on March 26, 2021

I have the same question. That is what I did when answering the question. I was able to eliminate the answer choice but I want to know if that isn't allowed.

Emil on January 25 at 08:50PM

We could have m in slot 1 if we have two ms and one I.